Nierówności I -- średnie dla starszych PDF Drukuj Email
Zadania II
Wpisany przez Joachim Jelisiejew   
wtorek, 25 października 2011 18:28

Zadania 
Zadania PDF.

Źródło zadań w texu.

 
%        File: zad.tex
%     Created: Sun Oct 23 03:00 PM 2011 C
% Last Change: Sun Oct 23 03:00 PM 2011 C
\documentclass[10pt]{article}
\usepackage{amssymb}
\usepackage{amsmath}
\usepackage{amsthm}
\textwidth 16cm
\textheight 24cm
\oddsidemargin 0cm
\topmargin 0pt
\headheight 0pt
\headsep 0pt
\usepackage[polish]{babel}
\usepackage[utf8]{inputenc}
\usepackage[T1]{fontenc}
\usepackage{polski}
\usepackage{import}
%\usepackage{MnSymbol}
% ----------------------------------------------------------------
\vfuzz4pt % Don't report over-full v-boxes if over-edge is small
\hfuzz4pt % Don't report over-full h-boxes if over-edge is small
% THEOREMS -------------------------------------------------------
\newtheorem{thm}{Twierdzenie}[section]
\newtheorem{cor}[thm]{Wniosek}
\newtheorem{lem}[thm]{Lemat}
\newtheorem{defn}[thm]{Definicja}
\newtheorem{tozs}[thm]{Tożsamość}
\newtheorem{hyp}[thm]{Hipoteza}
\newtheorem{useless}[thm]{}
 
\newenvironment{sol}[1][Rozwiązanie. ]{
\vskip 3mm
\noindent\emph{#1}
 
}
{\hfill\par}
 
\def\labelproblem{\sectionID{}\theproblem{}}
\newcounter{problem}
\newenvironment{problem}[1][Zadanie]{
\stepcounter{problem}
\vskip 3mm
\noindent{\textsc{\bfseries #1 \labelproblem}}\\}
{\hfill\par}
 
\def\abs #1{\left\vert #1\right\vert}
 
\renewcommand{\angle}{\sphericalangle}
\renewcommand{\vec}[1]{\overrightarrow{#1}}
\renewcommand{\leq}{\leqslant}
\renewcommand{\geq}{\geqslant}
\renewcommand{\dots}{\ldots}
 
%\subimport{../}{style.sty}
\subimport{./}{style-poprawki.sty}
%\include{style}
 
\def\headpicture{pi_roger.jpg}
\def\author{Joachim Jelisiejew}
\def\date{25 października 2011}
 
\def\bareroger{\includegraphics[height=1em]{jolly-roger-mat}}
\def\roger{\ \ \hbox{\bareroger{}}\ \ }
\usepackage{multicol}
\begin{document}
\section{Nierówności I}
 
\setcounter{subsection}{1}
\subsection{Starsi}
\def\sectionID{S}
 
\emph{Test na intuicję: wstaw znaki w~jak największej ilości poniższych
podpunktów w~ciągu 15 min.~Sprawdź, ile Twoich odpowiedzi było prawdziwych.
W~przypadku, gdy nie umiesz wybrać jednej opcji, wybierz dwie.}
\begin{problem}
    Wszystkie zmienne w~nierównościach są rzeczywiste dodatnie.
 
    We wszystkich poniższych nierównościach zastąp \bareroger{} jednym ze
    znaków: $\geq, \leq, <>$.
 
    \begin{itemize}
        \item Wstaw $\geq$ lub $\leq$, jeżeli uważasz, że otrzymana w~ten sposób
    nierówność jest prawdziwa.
 
        \item Wstaw $<>$ jeżeli uważasz, że zarówno po wstawieniu $\geq$ jak i~$\leq$
            otrzymamy nieprawdziwą nierówność.
    \end{itemize}
 
    Uzasadnij odpowiedź.
 
\begin{multicols}{2}
\begin{enumerate}
    \item $\frac{1}{a^3} + \frac{1}{b^3} + \frac{1}{c^3} \roger
        \frac{9}{a^3 + b^3 + c^3}$
    \item $\frac{4}{3}\left(a^2 + b^2 + c^2 \right) + 2 \roger 2(a + b + c)$
    \item $\frac{2}{a} + \frac{2}{b} + \frac{2}{c} \roger
        \frac{a+c}{b^2} + \frac{b+a}{c^2} + \frac{c+b}{a^2}$
    \item $2(a + b + c) \roger \frac{4bc}{b+c} + \frac{4ca}{a + c} +
        \frac{4ab}{a+b}$
    \item $2\left( a + b + c \right) \roger
        \frac{4a^2}{b + c} + \frac{4b^2}{a + c} + \frac{4c^2}{a + b}$
    \item $a^3b + b^3c + c^3d + d^3a \roger 4abcd$
\end{enumerate}
\end{multicols}
\begin{enumerate}
    \setcounter{enumi}{6}
    \item $a^3b + b^3c + c^3d + d^3a \roger a^2dc + b^2ad + c^2ba + d^2cb$
    \item Jeżeli $abcd=1$ to $\left(\frac{1}{a^2} + \frac{1}{b^2} + \frac{1}{c^2} +
        \frac{1}{d^2}\right)\left(a^2 + b^2 + c^2 + d^2\right) \roger 16$
    \item Jeżeli $abcd=1$ to $\frac{1}{a^2} + \frac{1}{b^2} + \frac{1}{c^2} +
        \frac{1}{d^2} \roger a^2 + b^2 + c^2 + d^2$
    \item $3\left(a^3 + b^3 + c^3\right) + \frac{1}{a} + \frac{1}{b} +
        \frac{1}{c} \roger 2\big(a(b+c) + b(a+c) + c(a+b)\big)$
\end{enumerate}
\end{problem}
 
\emph{K.K. (rozwinięcie na kółku)}
 
\begin{problem}
    Niech $a_1,\dots,a_n$ będą dodatnie, niech $S:=a_1 + a_2 + \dots + a_n$.
    Udowodnić, że
    \[
    \sum_{i=1}^n \frac{a_i}{S-a_i} \geq \frac{n}{n-1}.
    \]
\end{problem}
\begin{problem}
    Niech $a, b\in \left(0, \frac{1}{2}\right)$. Udowodnić, że
    \[
    \sqrt[2011]{ab} + \sqrt[2011]{b(1-a-b)} + \sqrt[2011]{(1-a-b)a} \leq
    \sqrt[2011]{3^{2009}}.
    \]
\end{problem}
 
\begin{problem}
    Liczby $a_1,\dots,a_n$ są dodatnie. Udowodnij, że
    \[
    \prod_{i=1}^n (1 + a_i) \leq \sum_{i=0}^n \frac{\left( a_1 + \dots + a_n
    \right)^i}{i!}.
    \]
\end{problem}
 
\end{document}
 
Poprawiony: wtorek, 25 października 2011 18:33